Last visit was: 26 Apr 2024, 00:02 It is currently 26 Apr 2024, 00:02

Close
GMAT Club Daily Prep
Thank you for using the timer - this advanced tool can estimate your performance and suggest more practice questions. We have subscribed you to Daily Prep Questions via email.

Customized
for You

we will pick new questions that match your level based on your Timer History

Track
Your Progress

every week, we’ll send you an estimated GMAT score based on your performance

Practice
Pays

we will pick new questions that match your level based on your Timer History
Not interested in getting valuable practice questions and articles delivered to your email? No problem, unsubscribe here.
Close
Request Expert Reply
Confirm Cancel
Tags:
Show Tags
Hide Tags
Manager
Manager
Joined: 24 Mar 2010
Posts: 57
Own Kudos [?]: 384 [0]
Given Kudos: 105
Send PM
e-GMAT Representative
Joined: 02 Nov 2011
Posts: 4348
Own Kudos [?]: 30797 [1]
Given Kudos: 635
GMAT Date: 08-19-2020
Send PM
e-GMAT Representative
Joined: 02 Nov 2011
Posts: 4348
Own Kudos [?]: 30797 [0]
Given Kudos: 635
GMAT Date: 08-19-2020
Send PM
avatar
Manager
Manager
Joined: 27 Jul 2012
Posts: 79
Own Kudos [?]: 203 [0]
Given Kudos: 101
Send PM
Re: Alternate Cause - A weakener or not [#permalink]
egmat wrote:
Another interesting official question:

A drug that is highly effective in treating many types of infection can, at present, be obtained only from the bark of the ibora, a tree that is quite rare in the wild. It takes the bark of 5,000 trees to make one kilogram of the drug. It follows, therefore, that continued production of the drug must inevitably lead to the ibora’s extinction.

Which of the following, if true, most seriously weakens the argument above?

(A) The drug made from ibora bark is dispensed to doctors from a central authority.
(B) The drug made from ibora bark is expensive to produce.
(C) The leaves of the ibora are used in a number of medical products.
(D) The ibora can be propagated from cuttings and grown under cultivation.
(E) The ibora generally grows in largely inaccessible places.

Let's see how many get this right :)

-Chiranjeev



I'll go for D.
Con: continued production of the drug must inevitably lead to the ibora’s extinction.
continued production------>Ibora's extinction
if we show that continued production does not necessarily lead to ibora's extinction the conclusion will fall apart.
D does exactly the same. it says that Ibora can grown under cultivation therefore making the drug out of Ibora's bark does not lead to Ibor's extinction, since cultivated Ibora can be used instead of natural one.
e-GMAT Representative
Joined: 02 Nov 2011
Posts: 4348
Own Kudos [?]: 30797 [0]
Given Kudos: 635
GMAT Date: 08-19-2020
Send PM
Re: Alternate Cause - A weakener or not [#permalink]
Expert Reply
Bibinaz wrote:
egmat wrote:
Another interesting official question:

A drug that is highly effective in treating many types of infection can, at present, be obtained only from the bark of the ibora, a tree that is quite rare in the wild. It takes the bark of 5,000 trees to make one kilogram of the drug. It follows, therefore, that continued production of the drug must inevitably lead to the ibora’s extinction.

Which of the following, if true, most seriously weakens the argument above?

(A) The drug made from ibora bark is dispensed to doctors from a central authority.
(B) The drug made from ibora bark is expensive to produce.
(C) The leaves of the ibora are used in a number of medical products.
(D) The ibora can be propagated from cuttings and grown under cultivation.
(E) The ibora generally grows in largely inaccessible places.

Let's see how many get this right :)

-Chiranjeev



I'll go for D.
Con: continued production of the drug must inevitably lead to the ibora’s extinction.
continued production------>Ibora's extinction
if we show that continued production does not necessarily lead to ibora's extinction the conclusion will fall apart.
D does exactly the same. it says that Ibora can grown under cultivation therefore making the drug out of Ibora's bark does not lead to Ibor's extinction, since cultivated Ibora can be used instead of natural one.


Hi Bibinaz,

You are absolutely correct.

Sorry for the late reply. I forgot about this completely.

Thanks,
Chiranjeev
avatar
Intern
Intern
Joined: 24 Jun 2013
Posts: 37
Own Kudos [?]: 19 [0]
Given Kudos: 49
Schools: ISB '16 NUS '15
Send PM
Re: Alternate Cause - A weakener or not [#permalink]
Hi Chiranjeev,

Just a short query on the alternative clause, What would be the weakener for X does not leads to Y ?

Thanks Nitin
e-GMAT Representative
Joined: 02 Nov 2011
Posts: 4348
Own Kudos [?]: 30797 [2]
Given Kudos: 635
GMAT Date: 08-19-2020
Send PM
Re: Alternate Cause - A weakener or not [#permalink]
2
Bookmarks
Expert Reply
Nitinaka19 wrote:
Hi Chiranjeev,

Just a short query on the alternative clause, What would be the weakener for X does not leads to Y ?

Thanks Nitin


Hi Nitin,

Think about it. Any statement that suggests "X leads to Y" would be a valid weakener for "X does not lead to Y". For example:

1. If a statement says that X leads to Z and Z leads to X. In such a case, indirectly though, X leads to Y. Hence, this statement will be a valid weakener.
2. Also, you can weaken this conclusion "X does not lead to Y" not by directly attacking the conclusion but by challenging the assumption or the logic of the argument. For example: If the argument relies on some study results, you can weaken the conclusion by saying that the study was actually biased.

We can think of even more ways to weaken "X does not lead to Y".

Does it address your query?

Thanks,
Chiranjeev
avatar
Intern
Intern
Joined: 26 Jan 2013
Posts: 4
Own Kudos [?]: [0]
Given Kudos: 1
Send PM
Re: Alternate Cause - A weakener or not [#permalink]
Hi,
I have read your article but was not able to understand following point:

alternate-cause-a-weakener-or-not-155034.html

The argument 2 is of the type X can lead to Y (X: Mismatched sleeping and waking cycles, Y: jeopardizing the marriage). So, this argument is talking about a generic case that X can lead to Y. Remember, in this argument, Y has not occurred in the past and the argument is not trying to explain the reasons for its occurrence.

Here you are saying the argument is not trying to explain the reasons for its occurrence, but in each of your example there is reason of occurrence.
Ex given by you :
12 years ago and again 5 years ago, there were extended periods when the Darfir Republic's currency, the pundra, was weak: its value was unusually low relative to the world's most stable currencies. Both times a weak pundra made Darfir's manufactured products a bargain on world markets, and Darfir's exports were up substantially. Now some politicians are saying that, in order to cause another similarly sized increase in exports, the government should allow the pundra to become weak again.

Here conclusion is week pundra will lead to similar size increase in export.
According to me reason is: a weak pundra made Darfir's manufactured products a bargain on world markets. thus reason for occurance is already in the argument. similarly we can find reason in other arguments too.

I did not understand why you said the argument is not trying to explain the reasons for its occurrence? Please highlight this.
e-GMAT Representative
Joined: 02 Nov 2011
Posts: 4348
Own Kudos [?]: 30797 [0]
Given Kudos: 635
GMAT Date: 08-19-2020
Send PM
Re: Alternate Cause - A weakener or not [#permalink]
Expert Reply
Quote:
Hi,
I have read your article but was not able to understand following point:

alternate-cause-a-weakener-or-not-155034.html

The argument 2 is of the type X can lead to Y (X: Mismatched sleeping and waking cycles, Y: jeopardizing the marriage). So, this argument is talking about a generic case that X can lead to Y. Remember, in this argument, Y has not occurred in the past and the argument is not trying to explain the reasons for its occurrence.

Here you are saying the argument is not trying to explain the reasons for its occurrence, but in each of your example there is reason of occurrence.
Ex given by you :
12 years ago and again 5 years ago, there were extended periods when the Darfir Republic's currency, the pundra, was weak: its value was unusually low relative to the world's most stable currencies. Both times a weak pundra made Darfir's manufactured products a bargain on world markets, and Darfir's exports were up substantially. Now some politicians are saying that, in order to cause another similarly sized increase in exports, the government should allow the pundra to become weak again.


Dear Karan,

Thank you for your post :) .
In order to productively respond to your query, I would like to request you to kindly fully address the point that you want to emphasize in argument 2. In your post, you point toward the second argument but then go on to talking about the third argument for reference. Unfortunately, the link between the two does not come out as clearly as you may have intended. In addition to this, could you also please share your understanding of the word reason with respect to causal arguments.

Look forward to your response :).
e-GMAT Representative
Joined: 02 Nov 2011
Posts: 4348
Own Kudos [?]: 30797 [1]
Given Kudos: 635
GMAT Date: 08-19-2020
Send PM
Re: Alternate Cause - A weakener or not [#permalink]
1
Kudos
Expert Reply
Hello Everyone,

Just wanted to share that we have posted solutions to 20 of the hardest official CR questions on our blog. We hope you learn from and enjoy them.

Here's the link:

https://e-gmat.com/blogs/?cat=21

If you like them or have any feedback, please feel free to post on this thread :)

All the best for your preparations :)

Regards,
Chiranjeev
Intern
Intern
Joined: 27 Aug 2014
Posts: 47
Own Kudos [?]: 22 [0]
Given Kudos: 3
Send PM
Re: Alternate Cause - A weakener or not [#permalink]
egmat wrote:
Almost all of us would have heard of causal arguments and most of us would also know the common weakener categories for such arguments. For example: one of the common weakeners is the one which suggests an alternate cause for the effect. Still, at times, we find that these common weakener categories don’t work.

The purpose of this article is to understand where these weakener categories don’t work and find out why.

EXERCISE

Before we begin, here is a small exercise for you consisting of three OG questions. Here, you have the question along with only one option statement and you need to find out if that option statement is a valid answer or not. A diligent attempt at the quiz will help you get the maximum out of this article.

1. Journalist: In physics journals, the number of articles reporting the results of experiments involving particle accelerators was lower last year than it had been in previous years. Several of the particle accelerators at major research institutions were out of service the year before last for repairs, so it is likely that the low number of articles was due to the decline in availability of particle accelerators.

Which of the following, if true, most seriously undermines the journalist’s argument?

Recent changes in the editorial policies of several physics journals have decreased the likelihood that articles concerning particle-accelerator research will be accepted for publication.

2. A study of marital relationships in which one partner's sleeping and waking cycles differ from those of other partner reveals that such couples share fewer activities with each other and have more violent arguments than do couples in a relationship in which both partners follow the same sleeping and waking patterns . Thus, mismatched sleeping and waking cycles can seriously jeopardize a marriage.

Which of the following, if true, most seriously weakens the argument above?

Married couples in which both spouses follow the same sleeping and waking patterns also have arguments that can jeopardize the couple's marriage.

3. 12 years ago and again 5 years ago, there were extended periods when the Darfir Republic's currency, the pundra, was weak: its value was unusually low relative to the world's most stable currencies. Both times a weak pundra made Darfir's manufactured products a bargain on world markets, and Darfir's exports were up substantially. Now some politicians are saying that, in order to cause another similarly sized increase in exports, the government should allow the pundra to become weak again.

Which of the following, if true, provides the government with the strongest grounds to doubt that the politicians' recommendation, if followed, will achieve its aim?

A sharp improvement in the efficiency of Darfir's manufacturing plants would make Darfir's product a bargain on world markets even without any weakening of the pundra relative to other currencies.


The answer for the quiz is that only in the first argument is the given option statement a valid answer. If you did all the three questions correctly, good job :)

If, in either question 2 or 3, you marked the option statement as a valid answer choice or found it very attractive, this article will help you understand where you went wrong and why.



UNDERSTANDING THE CONCLUSION

Let’s look back at three arguments and find out their conclusion statements.



Here, I’ll just reword each of the conclusion statements, without obviously changing their meaning, so that we can use them directly for our analysis.

The three conclusion statements can be rewritten as:



If we look at the conclusion statements carefully, we’ll observe that:

1. The first conclusion is of the type: X led to Y. In this conclusion type, we are trying to tell the reason which led to the occurrence of Y. So, obviously Y, an event or occurrence or process, has happened in the past and X, which we say led to Y, must also have happened in the past and before Y occurred.

2. The second and the third conclusions are of the type: X can/will lead to Y. Unlike the first type, here we are not explaining the reason for something that happened in the past. Y may or may not have happened in the past. In this conclusion, we are either presenting a generic case that X can lead to Y or a future prediction that X will lead to Y. The reason for clubbing these categories will become clear as we go through the article.

Now, let’s identify the elements X and Y for each of the conclusion statements:

In the first conclusion, we have



Let’s look at the option statements for these arguments:



As we look at each of these option statements, we see that what each of these is saying that there is an alternate cause/way, say Z, to achieve Y (the effect).



Now, when we look at these option statements and find that there is a Z which also leads to Y, we think that this existence of Z weakens both the conclusion types i.e. X led to Y and X can/will lead to Y. However, as the solutions to the OG questions tell us, that is not correct. Let’s understand this.

EXAMPLE

Let’s consider a simple example in which the argument says that

Eating sugar leads to obesity.

Can we weaken this statement by saying that

Eating oil leads to obesity.

The answer is No.

Why? Because the author is not saying that only eating sugar leads to obesity. The author is only saying that eating sugar is one of the ways to get obese. Even from common understanding, we all know that both of these things i.e. eating sugar and eating oil lead to obesity. The fact that eating oil makes you obese has no impact on the likelihood of the fact that eating sugar leads to obesity. In other words, saying that there are multiple ways to achieve the same objective does not weaken the conclusion which only states one of the ways to reach the objective.



MODIFIED EXAMPLE

Now, Let’s look at a modified version of our simple example:

If the argument says that

Eating sugar made Jon obese.

Can we weaken my statement by saying that

Eating oil made Jon obese.

The answer here is Yes. In this argument, we are essentially talking about a specific case i.e. what made Jon obese. Here, it is given knowledge that Jon is obese, what the argument or the conclusion provides is a reason that made Jon obese.

Now, when we make the above statement that Eating oil made Jon obese, we are essentially countering what the argument said. We are essentially saying that eating sugar is not the reason, rather eating oil is. We are creating significant doubt on the truth value of the argument.

At this point, can you understand how our statement did not weaken the original argument but how our statement weakens the modified argument?

The reason is that in the original argument i.e. Eating Sugar leads to obesity is a generic statement that X leads to Y. In such case, saying Z leads to Y does not impact the validity of the argument.
However, in the modified argument, we are talking about a very specific event (Y: obesity of Jon) and trying to explain the reason for the same (X: Eating Sugar). In this case, saying that there is some Z (eating oil) that led to Y weakens the argument because it creates significant doubts on whether eating sugar was the reason or not.

EXAMPLE 2

Let’s take one more example to understand this:

If the argument says that Pollution can cause cancer, then we cannot weaken this statement by saying that UV rays can cause cancer. The fact that UV rays can cause cancer has no impact on the statement that pollution can cause cancer.

However, if the argument says that Joe got cancer because of pollution, then we can definitely weaken the argument by suggesting that Joe got cancer because of exposure to UV rays. This is so because here, we are trying to explain the reason for a specific event i.e. Joe getting cancer. The argument says that the reason is pollution and when we say that the reason is UV rays, we are countering and thus, weakening the argument.

REVISITING EXERCISE ARGUMENTS

With the above understanding in mind, let’s bring back the exercise arguments and see if our understanding works there or not.



ARGUMENT 1
We can see that the first argument is of the type: X led to Y (X: Decline in availability of particle accelerators, Y: low number of articles). So, the argument is trying to explain the reason which led to the occurrence of Y. The argument is talking about a specific case in the past. It says that decline in availability of particle accelerators led to the low number of particles.

The option statement for this argument says that:

Recent changes in the editorial policies of several physics journals have decreased the likelihood that articles concerning particle-accelerator research will be accepted for publication.

This statement presents an alternate cause, Z, which could have led to the low number of articles. So, what this option statement is suggesting is that probably the actual reason for low number of articles is recent changes in the editorial policies of physics journals. By suggesting this, this creates doubt and hence weakens the argument which attributed the reason to decline in availability of particle accelerators.

ARGUMENT 2
The argument 2 is of the type X can lead to Y (X: Mismatched sleeping and waking cycles, Y: jeopardizing the marriage). So, this argument is talking about a generic case that X can lead to Y. Remember, in this argument, Y has not occurred in the past and the argument is not trying to explain the reasons for its occurrence. The argument is making a generic statement that X can lead to Y.

The option statement for this argument says that:

Married couples in which both spouses follow the same sleeping and waking patterns also have arguments that can jeopardize the couple's marriage.

The statement presents an alternate route, Z, to reach the same end Y (Z: arguments, Y: jeopardizing the marriage). But just the presence of one more way to reach the end does not weaken the original argument that X can lead to Y. The argument does not say that X (mismatched sleeping and waking cycles) is the only way to Y (jeopardize the marriage). If there are other ways to do so, it does not impact the argument.



ARGUMENT 3
The argument 3 is of the type: X will lead to Y (X: Weak pundra, Y: similarly sized increase in exports) So, this argument is talking about a future case that X will lead to Y. Remember, in this argument, Y has not occurred in the past and the argument is not trying to explain the reasons for its occurrence. The argument is making a futuristic statement that X will lead to Y.

The option statement for this argument says that:

A sharp improvement in the efficiency of Darfir's manufacturing plants would make Darfir's product a bargain on world markets even without any weakening of the pundra relative to other currencies

The statement presents an alternate route, Z, to reach the same end Y ( Z: A sharp improvement in the efficiency of Darfir's manufacturing plants, Y: similar sized increase in exports). Here again, just the presence of one more way to reach the end does not weaken the argument that X will lead to Y. The argument does not say that only X will lead to Y. If there are other ways to reach Y, it does not impact the argument.

TAKE AWAYS

1. “X leads/can lead/will lead to Y” allows the possibility of an alternate route, Z, to reach the effect, Y. Therefore, an option statement presenting an alternate route does not weaken this conclusion type.
2. “X led to Y” is presenting a reason (X) for a specific occurrence in the past (Y). An option statement suggesting an alternate cause, Z, which led to Y, creates doubts on the conclusion and thus, weakens the argument.


Hope this article clarifies some doubts.

Thank you :)
Chiranjeev Singh


Hi

Your article was very useful. Was wondering if the same principle helps for strengthening questions too? If yes, can you please detail
Manager
Manager
Joined: 24 Oct 2016
Posts: 196
Own Kudos [?]: 63 [0]
Given Kudos: 89
Location: India
Concentration: Finance, International Business
Schools: IIMB
GMAT 1: 550 Q42 V28
GPA: 3.96
WE:Human Resources (Retail Banking)
Send PM
Re: Alternate Cause - A weakener or not [#permalink]
to be simply wowwww thanks for the article mr. chirnajeev
Intern
Intern
Joined: 05 May 2016
Status:Preparing
Posts: 48
Own Kudos [?]: 20 [0]
Given Kudos: 150
Location: India
Concentration: International Business, Finance
Send PM
Re: Alternate Cause - A weakener or not [#permalink]
egmat wrote:
Almost all of us would have heard of causal arguments and most of us would also know the common weakener categories for such arguments. For example: one of the common weakeners is the one which suggests an alternate cause for the effect. Still, at times, we find that these common weakener categories don’t work.

The purpose of this article is to understand where these weakener categories don’t work and find out why.

EXERCISE

Before we begin, here is a small exercise for you consisting of three OG questions. Here, you have the question along with only one option statement and you need to find out if that option statement is a valid answer or not. A diligent attempt at the quiz will help you get the maximum out of this article.

1. Journalist: In physics journals, the number of articles reporting the results of experiments involving particle accelerators was lower last year than it had been in previous years. Several of the particle accelerators at major research institutions were out of service the year before last for repairs, so it is likely that the low number of articles was due to the decline in availability of particle accelerators.

Which of the following, if true, most seriously undermines the journalist’s argument?

Recent changes in the editorial policies of several physics journals have decreased the likelihood that articles concerning particle-accelerator research will be accepted for publication.

2. A study of marital relationships in which one partner's sleeping and waking cycles differ from those of other partner reveals that such couples share fewer activities with each other and have more violent arguments than do couples in a relationship in which both partners follow the same sleeping and waking patterns . Thus, mismatched sleeping and waking cycles can seriously jeopardize a marriage.

Which of the following, if true, most seriously weakens the argument above?

Married couples in which both spouses follow the same sleeping and waking patterns also have arguments that can jeopardize the couple's marriage.

3. 12 years ago and again 5 years ago, there were extended periods when the Darfir Republic's currency, the pundra, was weak: its value was unusually low relative to the world's most stable currencies. Both times a weak pundra made Darfir's manufactured products a bargain on world markets, and Darfir's exports were up substantially. Now some politicians are saying that, in order to cause another similarly sized increase in exports, the government should allow the pundra to become weak again.

Which of the following, if true, provides the government with the strongest grounds to doubt that the politicians' recommendation, if followed, will achieve its aim?

A sharp improvement in the efficiency of Darfir's manufacturing plants would make Darfir's product a bargain on world markets even without any weakening of the pundra relative to other currencies.


The answer for the quiz is that only in the first argument is the given option statement a valid answer. If you did all the three questions correctly, good job :)

If, in either question 2 or 3, you marked the option statement as a valid answer choice or found it very attractive, this article will help you understand where you went wrong and why.



UNDERSTANDING THE CONCLUSION

Let’s look back at three arguments and find out their conclusion statements.



Here, I’ll just reword each of the conclusion statements, without obviously changing their meaning, so that we can use them directly for our analysis.

The three conclusion statements can be rewritten as:



If we look at the conclusion statements carefully, we’ll observe that:

1. The first conclusion is of the type: X led to Y. In this conclusion type, we are trying to tell the reason which led to the occurrence of Y. So, obviously Y, an event or occurrence or process, has happened in the past and X, which we say led to Y, must also have happened in the past and before Y occurred.

2. The second and the third conclusions are of the type: X can/will lead to Y. Unlike the first type, here we are not explaining the reason for something that happened in the past. Y may or may not have happened in the past. In this conclusion, we are either presenting a generic case that X can lead to Y or a future prediction that X will lead to Y. The reason for clubbing these categories will become clear as we go through the article.

Now, let’s identify the elements X and Y for each of the conclusion statements:

In the first conclusion, we have



Let’s look at the option statements for these arguments:



As we look at each of these option statements, we see that what each of these is saying that there is an alternate cause/way, say Z, to achieve Y (the effect).



Now, when we look at these option statements and find that there is a Z which also leads to Y, we think that this existence of Z weakens both the conclusion types i.e. X led to Y and X can/will lead to Y. However, as the solutions to the OG questions tell us, that is not correct. Let’s understand this.

EXAMPLE

Let’s consider a simple example in which the argument says that

Eating sugar leads to obesity.

Can we weaken this statement by saying that

Eating oil leads to obesity.

The answer is No.

Why? Because the author is not saying that only eating sugar leads to obesity. The author is only saying that eating sugar is one of the ways to get obese. Even from common understanding, we all know that both of these things i.e. eating sugar and eating oil lead to obesity. The fact that eating oil makes you obese has no impact on the likelihood of the fact that eating sugar leads to obesity. In other words, saying that there are multiple ways to achieve the same objective does not weaken the conclusion which only states one of the ways to reach the objective.



MODIFIED EXAMPLE

Now, Let’s look at a modified version of our simple example:

If the argument says that

Eating sugar made Jon obese.

Can we weaken my statement by saying that

Eating oil made Jon obese.

The answer here is Yes. In this argument, we are essentially talking about a specific case i.e. what made Jon obese. Here, it is given knowledge that Jon is obese, what the argument or the conclusion provides is a reason that made Jon obese.

Now, when we make the above statement that Eating oil made Jon obese, we are essentially countering what the argument said. We are essentially saying that eating sugar is not the reason, rather eating oil is. We are creating significant doubt on the truth value of the argument.

At this point, can you understand how our statement did not weaken the original argument but how our statement weakens the modified argument?

The reason is that in the original argument i.e. Eating Sugar leads to obesity is a generic statement that X leads to Y. In such case, saying Z leads to Y does not impact the validity of the argument.
However, in the modified argument, we are talking about a very specific event (Y: obesity of Jon) and trying to explain the reason for the same (X: Eating Sugar). In this case, saying that there is some Z (eating oil) that led to Y weakens the argument because it creates significant doubts on whether eating sugar was the reason or not.

EXAMPLE 2

Let’s take one more example to understand this:

If the argument says that Pollution can cause cancer, then we cannot weaken this statement by saying that UV rays can cause cancer. The fact that UV rays can cause cancer has no impact on the statement that pollution can cause cancer.

However, if the argument says that Joe got cancer because of pollution, then we can definitely weaken the argument by suggesting that Joe got cancer because of exposure to UV rays. This is so because here, we are trying to explain the reason for a specific event i.e. Joe getting cancer. The argument says that the reason is pollution and when we say that the reason is UV rays, we are countering and thus, weakening the argument.

REVISITING EXERCISE ARGUMENTS

With the above understanding in mind, let’s bring back the exercise arguments and see if our understanding works there or not.



ARGUMENT 1
We can see that the first argument is of the type: X led to Y (X: Decline in availability of particle accelerators, Y: low number of articles). So, the argument is trying to explain the reason which led to the occurrence of Y. The argument is talking about a specific case in the past. It says that decline in availability of particle accelerators led to the low number of particles.

The option statement for this argument says that:

Recent changes in the editorial policies of several physics journals have decreased the likelihood that articles concerning particle-accelerator research will be accepted for publication.

This statement presents an alternate cause, Z, which could have led to the low number of articles. So, what this option statement is suggesting is that probably the actual reason for low number of articles is recent changes in the editorial policies of physics journals. By suggesting this, this creates doubt and hence weakens the argument which attributed the reason to decline in availability of particle accelerators.

ARGUMENT 2
The argument 2 is of the type X can lead to Y (X: Mismatched sleeping and waking cycles, Y: jeopardizing the marriage). So, this argument is talking about a generic case that X can lead to Y. Remember, in this argument, Y has not occurred in the past and the argument is not trying to explain the reasons for its occurrence. The argument is making a generic statement that X can lead to Y.

The option statement for this argument says that:

Married couples in which both spouses follow the same sleeping and waking patterns also have arguments that can jeopardize the couple's marriage.

The statement presents an alternate route, Z, to reach the same end Y (Z: arguments, Y: jeopardizing the marriage). But just the presence of one more way to reach the end does not weaken the original argument that X can lead to Y. The argument does not say that X (mismatched sleeping and waking cycles) is the only way to Y (jeopardize the marriage). If there are other ways to do so, it does not impact the argument.



ARGUMENT 3
The argument 3 is of the type: X will lead to Y (X: Weak pundra, Y: similarly sized increase in exports) So, this argument is talking about a future case that X will lead to Y. Remember, in this argument, Y has not occurred in the past and the argument is not trying to explain the reasons for its occurrence. The argument is making a futuristic statement that X will lead to Y.

The option statement for this argument says that:

A sharp improvement in the efficiency of Darfir's manufacturing plants would make Darfir's product a bargain on world markets even without any weakening of the pundra relative to other currencies

The statement presents an alternate route, Z, to reach the same end Y ( Z: A sharp improvement in the efficiency of Darfir's manufacturing plants, Y: similar sized increase in exports). Here again, just the presence of one more way to reach the end does not weaken the argument that X will lead to Y. The argument does not say that only X will lead to Y. If there are other ways to reach Y, it does not impact the argument.

TAKE AWAYS

1. “X leads/can lead/will lead to Y” allows the possibility of an alternate route, Z, to reach the effect, Y. Therefore, an option statement presenting an alternate route does not weaken this conclusion type.
2. “X led to Y” is presenting a reason (X) for a specific occurrence in the past (Y). An option statement suggesting an alternate cause, Z, which led to Y, creates doubts on the conclusion and thus, weakens the argument.


Hope this article clarifies some doubts.

Thank you :)
Chiranjeev Singh



Hi Chiranjeev.
Thanks for the article. I understood the point you wanted to convey.
I have one small doubt.In 1st argument the word several is there which i believe coveys that not all journals altered there policies. Can we say on this basis that the answer does not weaken the argument completely?
avatar
Intern
Intern
Joined: 24 Nov 2015
Posts: 1
Own Kudos [?]: [0]
Given Kudos: 11
Send PM
Re: Alternate Cause - A weakener or not [#permalink]
what happens in case of X leads to Y
in that case alternate cause should create enough doubt
please explain.
e-GMAT Representative
Joined: 02 Nov 2011
Posts: 4348
Own Kudos [?]: 30797 [1]
Given Kudos: 635
GMAT Date: 08-19-2020
Send PM
Alternate Cause - A weakener or not [#permalink]
1
Bookmarks
Expert Reply
gauravk123 – I would like to draw your attention to the TakeAway from the article.

Quote:
“X leads/can lead/will lead to Y” allows the possibility of an alternate route, Z, to reach the effect, Y. Therefore, an option statement presenting an alternate route does not weaken this conclusion type.


Hence, an alternate route (remember since its leads to, we call it route not cause) does not always weaken the conclusion.

Let’s take an example:

Google, a technology company, reported high profits yesterday and its stock price increased. Hence, reporting high profits leads to increase in stock price.


Now would the statement below weaken your belief in the conclusion

Vmware, a technology company, saw its stock price increase on speculation about its acquisition.

Absolutely not .. why because your original conclusion does not imply that reporting high profits is the “only” to increase a company’s stock price.

Now, I will add another twist to this .. if there were an answer choice that said..

Speculation about a company’s acquisition always leads to increase in company’s stock price and there was plenty of speculation yesterday about Apple acquiring Google


Now while such answer choices are very rare but if such a choice is presented, it would weaken the conclusion. Why – because its attacking the basis of author’s conclusion .. the reason why Google’s stock price increased.

The key thing you need to remember is to understand the author’s reasoning which is what we want you to take-away from this article.

I hope the above helps. Take a look at the assumption files in our free trial to learn more.

-Rajat
Intern
Intern
Joined: 23 Sep 2017
Posts: 15
Own Kudos [?]: 8 [0]
Given Kudos: 34
Send PM
Re: Alternate Cause - A weakener or not [#permalink]
Could you please clarify this question as, I believe, it belongs to this kind
In the last five years there has been a significant increase in the consumption of red wine. During this same period, there have been several major news reports about the beneficial long-term effects on health that certain antioxidants in red wine can provide. Thus, the increase in red wine consumption can be directly attributed to consumers’
recognition of the beneficial effects of antioxidants.

Which of the following, if true, most seriously undermines the explanation above?

(A) Sales of other alcoholic beverages have not increased in the last five years.
(B) On average, people consume about 10 percent more red wine than they did five years ago.
(C) The health benefits of red wine are usually not noticeable for several years.
(D) The consumption of grape juice and other antioxidant-rich products has also increased in the last five years.
(E) Red wine prices have decreased significantly in the last five years, while the prices of other alcoholic beverages have risen
steadily.

The correct answer is E, which presents the alternate cause for the increase in the consumption of red wine. However, the conclusion is just the generic prediction.
I was so confused.
IIM School Moderator
Joined: 04 Sep 2016
Posts: 1261
Own Kudos [?]: 1240 [1]
Given Kudos: 1207
Location: India
WE:Engineering (Other)
Send PM
Re: Alternate Cause - A weakener or not [#permalink]
1
Bookmarks
egmat Harshgmat nightblade354

Regarding the takeaways, can I simple recall below jist by remembering verb tenses ie present / past or future?
1. “X leads/can lead/will lead to Y” allows the possibility of an alternate route, Z, to reach the effect, Y. Therefore, an option statement presenting an alternate route does not weaken this conclusion type. Present / Future tense
2. “X led to Y” is presenting a reason (X) for a specific occurrence in the past (Y). An option statement suggesting an alternate cause, Z, which led to Y, creates doubts on the conclusion and thus, weakens the argument. Past tense
Intern
Intern
Joined: 23 Sep 2014
Posts: 21
Own Kudos [?]: 10 [0]
Given Kudos: 117
Send PM
Alternate Cause - A weakener or not [#permalink]
egmat wrote:
Hi Skamal7 and Vikas,

Thank you for the appreciation.

@Skamal7: You have cited a very relevant question here. Thank you.

As I look at your analysis, I can see that there are some gaps in the understanding. Let's try to understand these.

Firstly, 'would' does not indicate that the event happened in the past. "Removal of tariffs" has not happened; it may happen in the future. Refer to this page for different uses of "would": https://www.ecenglish.com/learnenglish/l ... -use-would

In the present context, the use of "would" is the third kind of use of "would" (conditional sentences) as described on this page.

Now, with the understanding that we are talking about a future event, not a past event, we are also clear that even if we replace "would" by "will", it will not make a difference.

However, then, why is option E the correct choice?

The answer is not that it provides an alternate cause. The answer is that it says that the current scenario is leading to urban unemployment and by doing this, it weakens the conclusion that the changed scenario (with tariffs removed) will lead to increased urban unemployment.

In terms of X and Y, the conclusion says that X will lead to Y (X: removal of tariffs, Y: urban unemployment).Option E weakens this by indicating that by not doing anything (i.e. with the status quo of high tariffs), we'll have more urban employment. This indicates that X will probably lead to reduction in Y, than increase in Y.
(Here, Y is not an event which will happen or not happen. It is a continuous figure which may increase or decrease).

Now, suppose, we were given an alternate cause/route of increasing urban unemployment:

A lack of funds available for training the unemployed youth in the urban areas is leading to their continued unemployment.

This option provides an alternate way for unemployment in urban areas i.e. lack of funds for training. Would this weaken the argument?

The answer is No. This is what the article tells.

The important point is to understand option E. It does not provide an alternate cause; it talks of the current scenario and suggests that X will possibly lead to reduction in Y, than an increase in Y.

Does this help?

Thanks :)
Chiranjeev


Hi egmat,

I would like to ask on this question on the formal logic approach

Kernland imposes a high tariff on the export of unprocessed cashew nuts in order to ensure that the nuts are sold to domestic processing plants. If the tariff were lifted and unprocessed cashews were sold at world market prices, more farmers could profit by growing cashews. However, since all the processing plants are in urban areas, removing the tariff would seriously hamper the government’s effort to reduce urban unemployment over the next five years.

Which of the following, if true, most seriously weakens the argument?

E. A lack of profitable crops is driving an increasing number of small farmers in Kernland off their land and into the cities
."

So in the stimulus, it is basically saying that With the current Tariff, we have more unprofitable crops which, as in option E states, that will increase the unemployment. This can be described as Less Profitable Crops --> Higher Unemployment (Less Profitable Crops is the sufficient condition)

As per your reasoning, option E weakens the argument because it says that by removing the tariff, the farmers will have more profitable crops, thus will lead to reduced unemployment as they dont have to leave their farms. This can be described as More Profitable Crops --> Lower Unemployment

However, in terms of formal logic, can we infer from the Less Profitable Crops --> Higher Unemployment to More Profitable Crops --> Lower Unemployment? If I understand formal logic correctly, in order to infer the sufficient conditioning like this, it has to be in form of Lower Unemployment --> More Profitable Crops (negate both elements and change their places between sufficient and necessary conditions), meaning we CANNOT infer that More Profitable Crops can lead to Lower Unemployment.

Hope you can clear my doubt on how can we infer from option E using the formal logic.
GMAT Club Bot
Alternate Cause - A weakener or not [#permalink]
   1   2   3 
Moderators:
GMAT Club Verbal Expert
6921 posts
GMAT Club Verbal Expert
238 posts
GRE Forum Moderator
13961 posts

Powered by phpBB © phpBB Group | Emoji artwork provided by EmojiOne